KVPY SA 2016

Algebra Level 3

The positive coefficients a , b , c a,b,c in the quadratic polynomial P ( x ) = a x 2 + b x + c P(x) =ax^{2} +bx +c form an arithmetic progression in that order.

If P ( x ) P(x) has integer roots P P and Q Q , then what is P + Q + P Q ? P+Q+PQ?

3 5 7 14

This section requires Javascript.
You are seeing this because something didn't load right. We suggest you, (a) try refreshing the page, (b) enabling javascript if it is disabled on your browser and, finally, (c) loading the non-javascript version of this page . We're sorry about the hassle.

1 solution

With a , b , c > 0 a,b,c \gt 0 in arithmetic progression, let b = a + k b = a + k and c = a + 2 k c = a + 2k for some real k k . Then by Vieta's we know that P + Q = b a = 1 k a P + Q = -\dfrac{b}{a} = -1 - \dfrac{k}{a} and that P Q = c a = 1 + 2 k a PQ = \dfrac{c}{a} = 1 + 2\dfrac{k}{a} , and thus P + Q + P Q = k a P + Q + PQ = \dfrac{k}{a} .

Now as P , Q P,Q must be integers we will require that k a \dfrac{k}{a} be an integer, and so k = n a k = na for some integer n n . It is the possible values of n n that we are then wanting to find. The given quadratic can then be written as

a x 2 + ( a + n a ) x + ( a + 2 n a ) = a ( x 2 + a ( n + 1 ) x + a ( 2 n + 1 ) ) = 0 x 2 + ( n + 1 ) x + ( 2 n + 1 ) = 0 ax^{2} + (a + na)x + (a + 2na) = a(x^{2} + a(n + 1)x + a(2n + 1)) = 0 \Longrightarrow x^{2} + (n + 1)x + (2n + 1) = 0

x = ( n + 1 ) ± ( n + 1 ) 2 4 ( 2 n + 1 ) 2 = ( n + 1 ) ± n 2 6 n 3 2 \Longrightarrow x = \dfrac{-(n + 1) \pm \sqrt{(n + 1)^{2} - 4(2n + 1)}}{2} = \dfrac{-(n + 1) \pm \sqrt{n^{2} - 6n - 3}}{2} ,

which will only have integer solutions if n 2 6 n 3 = m 2 n^{2} - 6n - 3 = m^{2} for some (positive) integer m m . Completing the square, we then have that

( n 3 ) 2 12 = m 2 ( n 3 ) 2 m 2 = 12 ( ( n 3 ) m ) ( ( n 3 ) + m ) = 12 (n - 3)^{2} - 12 = m^{2} \Longrightarrow (n - 3)^{2} - m^{2} = 12 \Longrightarrow ((n - 3) - m)((n - 3) + m) = 12 .

Now when 12 12 can be factored as a b ab , with a < b a \lt b , then we can assign a = ( n 3 ) m a = (n - 3) - m and b = ( n 3 ) + m b = (n - 3) + m , for which a + b = 2 ( n 3 ) a + b = 2(n - 3) , an even integer. So we need only consider the factorings of 12 12 where either a , b a,b are both even or are both odd. These are ( a , b ) = ( 2 , 6 ) (a,b) = (2,6) and ( a , b ) = ( 6 , 2 ) (a,b) = (-6,-2) , which give us values for n n of 7 7 and 1 -1 . With n = 1 n = -1 we would end up with b = 0 b = 0 , violating the condition that b b be positive, so we are left with P + Q + P Q = n = 7 P + Q + PQ = n = \boxed{7} .

For this value of n n we have that m = 2 m = 2 , and thus x = 8 ± 2 2 P = 3 , Q = 5 x = \dfrac{-8 \pm 2}{2} \Longrightarrow P = -3, Q = -5 . So for example, with a = 1 a = 1 we would have b = 8 b = 8 and c = 15 c = 15 , giving us the quadratic x 2 + 8 x + 15 = ( x + 3 ) ( x + 5 ) x^{2} + 8x + 15 = (x + 3)(x + 5) , which has integer roots P = 3 , Q = 5 P = -3, Q = -5 such that P + Q + P Q = 3 + ( 5 ) + ( 3 ) ( 5 ) = 7 P + Q + PQ = -3 + (-5) + (-3)(-5) = 7 as anticipated.

P+Q+PQ=3k/a not k/a

Uma Balan - 2 years, 6 months ago

Log in to reply

Good catch! My actual mistake though was that I had written P + Q = 1 + k a P + Q = -1 + \dfrac{k}{a} rather than the correct value of 1 k a -1 - \dfrac{k}{a} , so in the end P + Q + P Q = k a P + Q + PQ = \dfrac{k}{a} is correct.

Brian Charlesworth - 2 years, 6 months ago

0 pending reports

×

Problem Loading...

Note Loading...

Set Loading...